(6 intermediate revisions by 2 users not shown)
Line 20: Line 20:
  
 
<math> x(t)=e^{-3|t|}=e^{3t}u(-t)+e^{-3t}u(t) </math>.
 
<math> x(t)=e^{-3|t|}=e^{3t}u(-t)+e^{-3t}u(t) </math>.
 +
:<span style="color:red"> Instructor's comment: Actually, this is only true for <math>t\neq 0 </math>, since <math>x(0)=1</math>. However, changing the value of a signal at a single point does not change its Fourier transform. -pm  </span>
  
 
From the table:
 
From the table:
Line 52: Line 53:
  
 
<math> \frac{1}{2\pi} \int_{-\infty}^{\infty} e^{-j\omega t} dt = \delta(\omega) </math>
 
<math> \frac{1}{2\pi} \int_{-\infty}^{\infty} e^{-j\omega t} dt = \delta(\omega) </math>
 +
 +
:<span style="color:red"> Instructor's note: In order to use this property, one first needs to prove it. Unfortunately, proving it is quite tricky, as was noted in class. Therefore, the above method cannot be used in this homework. Here is a correct solution: </span>
 +
 +
:<math>
 +
\begin{align}
 +
\sin^2\left(\pi t +\frac{\pi}{12}\right)&= \left( \frac{1}{2j}e^{j (\pi t +\frac{\pi}{12})}-\frac{1}{2j}e^{-j (\pi t +\frac{\pi}{12})} \right)^2 \\
 +
&= \left( \frac{1}{2j}e^{j \frac{\pi}{12}}e^{j\pi t}-\frac{1}{2j}e^{-j \frac{\pi}{12}}e^{-j \pi t} \right)^2 \\
 +
& =  \frac{1}{2} - \frac{e^{j\frac{\pi}{6}}}{4}  e^{j 2\pi t}  -\frac{e^{-j\frac{\pi}{6}}}{4} e^{-j2\pi t}  \\
 +
\Rightarrow {\mathcal F} \left( \sin^2\left(\pi t +\frac{\pi}{12}\right)\right) &= \int_{-\infty}^\infty \left( \frac{1}{2} - \frac{e^{j\frac{\pi}{6}}}{4}  e^{j 2\pi t}  -\frac{e^{-j\frac{\pi}{6}}}{4} e^{-j2\pi t}  \right) e^{-j\omega t} dt \\
 +
&= \int_{-\infty}^\infty  \frac{1}{2}  e^{-j\omega t} dt - \int_{-\infty}^\infty \frac{e^{j\frac{\pi}{6}}}{4}  e^{j 2\pi t}  e^{-j\omega t} dt  -\int_{-\infty}^\infty \frac{e^{-j\frac{\pi}{6}}}{4} e^{-j2\pi t}  e^{-j\omega t} dt \\
 +
&=  \frac{1}{2} \int_{-\infty}^\infty  e^{-j\omega t} dt - \frac{e^{j\frac{\pi}{6}}}{4}  \int_{-\infty}^\infty  e^{j 2\pi t}  e^{-j\omega t} dt  -\frac{e^{-j\frac{\pi}{6}}}{4}\int_{-\infty}^\infty  e^{-j2\pi t}  e^{-j\omega t} dt \\
 +
&=  \frac{1}{2} {\mathcal F} \left(1\right) - \frac{e^{j\frac{\pi}{6}}}{4}  {\mathcal F} \left( e^{j 2\pi t} \right)  -\frac{e^{-j\frac{\pi}{6}}}{4} {\mathcal F} \left( e^{-j2\pi t} \right).
 +
\end{align}
 +
</math>
 +
:<span style="color:red">So all we need is the Fourier transform of <math class="inline">e^{j \omega_0 t}</math>, which cannot be found by integration. Since we are not allowed to use the table of properties and pairs, we will have to "guess it" and justify our answer.</span>
 +
:Claim: <math class="inline">  {\mathcal F} \left(e^{j \omega_0 t} \right) = 2 \pi \delta(\omega-\omega_0)</math>
 +
:Proof: <math class="inline">  {\mathcal F}^{-1} \left( 2 \pi \delta(\omega-\omega_0) \right) = \frac{1}{2\pi} \int_{-\infty}^{\infty} 2 \pi \delta(\omega-\omega_0) e^{-j\omega t} d\omega = e^{-j\omega_0 t} </math>.
 +
 +
:<span style="color:red">Therefore </span>
 +
:<math>
 +
{\mathcal F} \left( \sin^2\left(\pi t +\frac{\pi}{12}\right)\right)= \pi \delta(\omega) - \frac{\pi e^{j\frac{\pi}{6}}}{2} \delta(\omega - 2\pi) - \frac{\pi e^{-j\frac{\pi}{6}}}{2} \delta(\omega + 2\pi)
 +
</math>
 +
:<span style="color:red">-pm </span> 
 +
 +
  
 
Now to verify our answer we write:
 
Now to verify our answer we write:
Line 71: Line 97:
  
 
where we have used the fact that <math>f(t)\delta(t-t_0)=f(t_0)\delta(t-t_0)</math>.
 
where we have used the fact that <math>f(t)\delta(t-t_0)=f(t_0)\delta(t-t_0)</math>.
 +
 +
:<span style="color:red">Instructor's note: Alternatively, you can view x(t) as a product of two sinusoidal. By the multiplication property, the Fourier transform of x(t) is thus the convolution of the Fourier transform of a sinusoidal with itself. -pm </span>
  
 
==Question 3==
 
==Question 3==
Line 102: Line 130:
 
<math>\mathcal{Y}(\omega)=\frac{e^{-j\frac{2}{3}\omega}}{3}\mathcal{X}(-\frac{\omega}{3})</math>
 
<math>\mathcal{Y}(\omega)=\frac{e^{-j\frac{2}{3}\omega}}{3}\mathcal{X}(-\frac{\omega}{3})</math>
  
 +
 +
:<span style="color:red">Instructor's note: It is probably easier to do this problem using the definition of the Fourier transform. </span>
 +
:<math>
 +
\begin{align}
 +
{\mathcal F} \left( x(-3t+2) \right) &= \int_{-\infty}^\infty x(-3t+2) e^{-j\omega t} dt, \\
 +
&= \int_{\infty}^{-\infty} x(u) e^{-j\omega \frac{(u-2)}{-3}} \frac{du}{-3}, \text{ (letting }u=-3t+2), \\
 +
&= \frac{1}{3} \int_{-\infty}^\infty x(u)e^{-\frac{j\omega u}{-3}}e^{\frac{-2j\omega}{3}} du,\\
 +
&= \frac{e^{\frac{-2j\omega}{3}}}{3}\int_{-\infty}^\infty x(u)e^{-\frac{j\omega u}{-3}} du, \\
 +
&= \frac{e^{\frac{-2j\omega}{3}}}{3} \mathcal{X}(\frac{\omega}{-3}).
 +
\end{align}
 +
</math>
 
==Question 5==
 
==Question 5==
 
a)  
 
a)  
  
<math>\mathcal{H}(\omega)=\mathfrak{F}\left\{e^{-3t}u(t)\right\}=\frac{1}{3+j\omega}</math>
+
<math>\mathcal{H}(\omega)=\mathfrak{F}\left\{e^{-3t}u(t)\right\}=\int_{-\infty}^\infty e^{-3t}u(t) e^{-j\omega t} dt = \int_{0}^\infty e^{-3t} e^{-j\omega t} dt =\left. \frac{e^{(-3-j\omega) t}}{ -3 -j \omega }\right|_0^\infty =\frac{1}{3+j\omega}</math>
  
 
b)
 
b)
Line 159: Line 198:
 
The frequency response of the system can be written as:
 
The frequency response of the system can be written as:
  
<math>\mathcal{H}(\omega)=\frac{1}{j\omega-1}{j\omega-2}</math>.
+
<math>\mathcal{H}(\omega)=\frac{1}{(j\omega-1)(j\omega-2)}</math>.
  
 
Note that this can be done by either direct inspection or by finding the roots of the quadratic equation in the denominator.
 
Note that this can be done by either direct inspection or by finding the roots of the quadratic equation in the denominator.
Line 177: Line 216:
 
Then,
 
Then,
  
<math>\mathcal{H}(\omega)=-\frac{1}{j\omega-1}+\frac{1}{j\omega-2}</math>
+
<math>\mathcal{H}(\omega)=-\frac{1}{j\omega-1}+\frac{1}{j\omega-2}=\frac{1}{1-j\omega}-\frac{1}{2-j\omega}</math>
  
Now, using FT pairs, we have:
+
Now, using linearity of FT, FT pairs, and time reverse property, we get:
  
<math>h(t)=e^{2t}u(t)-e^tu(t)</math> .
+
<math>h(t)=e^{t}u(-t)-e^{2t}u(-t)</math> .
  
 
   
 
   

Latest revision as of 11:34, 7 March 2011

Homework 5 Solutions, ECE301 Spring 2011 Prof. Boutin

Students should feel free to make comments/corrections or ask questions directly on this page.

Question 1

$ \begin{align} \mathcal{X}(\omega)&= \int_{-\infty}^{\infty} e^{-3|t|}e^{-j\omega t} dt \\ &= \int_{-\infty}^0 e^{3t}e^{-j\omega t} dt + \int_0^{\infty} e^{-3t}e^{-j\omega t} dt \\ &= \int_{-\infty}^0 e^{(-j\omega +3)t} dt + \int_0^{\infty} e^{-(j\omega +3)t} dt \\ &= \frac{1}{-j\omega +3} \left[e^{(-j\omega +3)t}\right]_{-\infty}^0 - \frac{1}{j\omega +3} \left[e^{-(j\omega +3)t}\right]^{\infty}_0 \\ &= \frac{1}{-j\omega +3} \left[1-0 \right] - \frac{1}{j\omega +3} \left[0-1\right] \\ &= \frac{1}{-j\omega +3} + \frac{1}{j\omega +3} \\ &= \frac{j\omega +3}{\omega^2 +9}+\frac{-j\omega +3}{\omega^2 + 9} \\ &= \frac{6}{\omega^2+9} \end{align} $

To verify our answer using the table, we first write:

$ x(t)=e^{-3|t|}=e^{3t}u(-t)+e^{-3t}u(t) $.

Instructor's comment: Actually, this is only true for $ t\neq 0 $, since $ x(0)=1 $. However, changing the value of a signal at a single point does not change its Fourier transform. -pm

From the table:

$ \mathfrak{F}\left\{e^{-3t}u(t)\right\}=\frac{1}{j\omega+3} $.

Using time-reversal property from the table:

$ \mathfrak{F}\left\{e^{3t}u(-t)\right\}=\frac{1}{-j\omega+3} $.

Now, since the Fourier transform (FT) is linear, we have that:

$ \begin{align} \mathcal{X}(\omega)&=\mathfrak{F}\left\{e^{-3t}u(t)\right\}+\mathfrak{F}\left\{e^{3t}u(-t)\right\} \\ &=\frac{1}{j\omega+3}+\frac{1}{-j\omega+3} \\ &=\frac{6}{\omega^2+9} \end{align} $.


Question 2

$ \begin{align} \mathcal{X}(\omega)&= \int_{-\infty}^{\infty} \sin^2\left(\pi t +\frac{\pi}{12}\right)e^{-j\omega t}dt \\ &= \int_{-\infty}^{\infty} \left[\frac{1}{2} - \frac{1}{2}\cos\left(2\pi t + \frac{\pi}{6}\right) e^{-j\omega t} \right]dt \\ &= \frac{1}{2} \int_{-\infty}^{\infty} e^{-j\omega t} dt - \frac{e^{j\frac{\pi}{6}}}{4} \int_{-\infty}^{\infty} e^{j2\pi t} e^{-j\omega t} dt -\frac{e^{-j\frac{\pi}{6}}}{4} \int_{-\infty}^{\infty}e^{-j2\pi t}e^{-j\omega t} dt \\ &= \frac{1}{2} \int_{-\infty}^{\infty} e^{-j\omega t} dt - \frac{e^{j\frac{\pi}{6}}}{4} \int_{-\infty}^{\infty} e^{j(2\pi -\omega)t} dt -\frac{e^{-j\frac{\pi}{6}}}{4} \int_{-\infty}^{\infty}e^{-j(2\pi + \omega)t} dt \\ &=\pi \delta(\omega) - \frac{\pi e^{j\frac{\pi}{6}}}{2} \delta(\omega - 2\pi) - \frac{\pi e^{-j\frac{\pi}{6}}}{2} \delta(\omega + 2\pi), \end{align} $

where we have used the following property:

$ \frac{1}{2\pi} \int_{-\infty}^{\infty} e^{-j\omega t} dt = \delta(\omega) $

Instructor's note: In order to use this property, one first needs to prove it. Unfortunately, proving it is quite tricky, as was noted in class. Therefore, the above method cannot be used in this homework. Here is a correct solution:
$ \begin{align} \sin^2\left(\pi t +\frac{\pi}{12}\right)&= \left( \frac{1}{2j}e^{j (\pi t +\frac{\pi}{12})}-\frac{1}{2j}e^{-j (\pi t +\frac{\pi}{12})} \right)^2 \\ &= \left( \frac{1}{2j}e^{j \frac{\pi}{12}}e^{j\pi t}-\frac{1}{2j}e^{-j \frac{\pi}{12}}e^{-j \pi t} \right)^2 \\ & = \frac{1}{2} - \frac{e^{j\frac{\pi}{6}}}{4} e^{j 2\pi t} -\frac{e^{-j\frac{\pi}{6}}}{4} e^{-j2\pi t} \\ \Rightarrow {\mathcal F} \left( \sin^2\left(\pi t +\frac{\pi}{12}\right)\right) &= \int_{-\infty}^\infty \left( \frac{1}{2} - \frac{e^{j\frac{\pi}{6}}}{4} e^{j 2\pi t} -\frac{e^{-j\frac{\pi}{6}}}{4} e^{-j2\pi t} \right) e^{-j\omega t} dt \\ &= \int_{-\infty}^\infty \frac{1}{2} e^{-j\omega t} dt - \int_{-\infty}^\infty \frac{e^{j\frac{\pi}{6}}}{4} e^{j 2\pi t} e^{-j\omega t} dt -\int_{-\infty}^\infty \frac{e^{-j\frac{\pi}{6}}}{4} e^{-j2\pi t} e^{-j\omega t} dt \\ &= \frac{1}{2} \int_{-\infty}^\infty e^{-j\omega t} dt - \frac{e^{j\frac{\pi}{6}}}{4} \int_{-\infty}^\infty e^{j 2\pi t} e^{-j\omega t} dt -\frac{e^{-j\frac{\pi}{6}}}{4}\int_{-\infty}^\infty e^{-j2\pi t} e^{-j\omega t} dt \\ &= \frac{1}{2} {\mathcal F} \left(1\right) - \frac{e^{j\frac{\pi}{6}}}{4} {\mathcal F} \left( e^{j 2\pi t} \right) -\frac{e^{-j\frac{\pi}{6}}}{4} {\mathcal F} \left( e^{-j2\pi t} \right). \end{align} $
So all we need is the Fourier transform of $ e^{j \omega_0 t} $, which cannot be found by integration. Since we are not allowed to use the table of properties and pairs, we will have to "guess it" and justify our answer.
Claim: $ {\mathcal F} \left(e^{j \omega_0 t} \right) = 2 \pi \delta(\omega-\omega_0) $
Proof: $ {\mathcal F}^{-1} \left( 2 \pi \delta(\omega-\omega_0) \right) = \frac{1}{2\pi} \int_{-\infty}^{\infty} 2 \pi \delta(\omega-\omega_0) e^{-j\omega t} d\omega = e^{-j\omega_0 t} $.
Therefore
$ {\mathcal F} \left( \sin^2\left(\pi t +\frac{\pi}{12}\right)\right)= \pi \delta(\omega) - \frac{\pi e^{j\frac{\pi}{6}}}{2} \delta(\omega - 2\pi) - \frac{\pi e^{-j\frac{\pi}{6}}}{2} \delta(\omega + 2\pi) $
-pm


Now to verify our answer we write:

$ \begin{align} x(t)&=\sin^2\left(\pi t - \frac{\pi}{12}\right) \\ &= \frac{1}{2}-\frac{1}{2}\cos\left(2\pi t + \frac{\pi}{6}\right) \\ &= \frac{1}{2}-\frac{1}{2}\cos\left[2\pi\left( t + \frac{1}{12}\right)\right] \end{align} $

Now using (from the table) the time-shift property, FT of a constant, and FT of a cosine we get:

$ \begin{align} \mathcal{X}(\omega) &= \pi\delta(\omega) - \frac{\pi e^{j\frac{\omega}{12}}}{2} \delta(\omega - 2\pi)- \frac{\pi e^{j\frac{\omega}{12}}}{2} \delta(\omega + 2\pi) \\ &= \pi\delta(\omega) - \frac{\pi e^{j\frac{\pi}{6}}}{2} \delta(\omega - 2\pi)- \frac{\pi e^{-j\frac{\pi}{6}}}{2} \delta(\omega + 2\pi) \end{align} $

where we have used the fact that $ f(t)\delta(t-t_0)=f(t_0)\delta(t-t_0) $.

Instructor's note: Alternatively, you can view x(t) as a product of two sinusoidal. By the multiplication property, the Fourier transform of x(t) is thus the convolution of the Fourier transform of a sinusoidal with itself. -pm

Question 3

$ \begin{align} E_{\infty} \left\{x(t)\right\} &= \frac{1}{2\pi} \int_{-\infty}^{\infty} \left| |\omega|^3 [u(\omega + 8)-u(\omega-5)]\right|^2 d\omega \\ &=\frac{1}{2\pi} \int_{-\infty}^{\infty} \omega^6[u(\omega+8)-u(\omega-5)]d\omega \\ &=\frac{1}{2\pi} \int_{-8}^{5}\omega^6 d\omega \\ &=\frac{1}{14\pi} \omega^7\Bigg|^5_{-8} \\ &=\frac{1}{14\pi}[5^7-(-8)^7] \\ &=\frac{2175277}{14\pi} \end{align} $

Question 4

$ \begin{align} y(t)&=x(-3t+2) \\ &=x\left[-3\left(t-\frac{2}{3}\right)\right] \end{align} $

From this we can deduce that we have time scaling first by a factor of -3, and then we have a time delay by 2/3.

Now,

$ \mathfrak{F}\left\{x(-3t)\right\}=\frac{1}{3}\mathcal{X}(-\frac{\omega}{3}) $,

using the time scaling property of the FT.

Then,

$ \mathcal{Y}(\omega)=\frac{e^{-j\frac{2}{3}\omega}}{3}\mathcal{X}(-\frac{\omega}{3}) $


Instructor's note: It is probably easier to do this problem using the definition of the Fourier transform.
$ \begin{align} {\mathcal F} \left( x(-3t+2) \right) &= \int_{-\infty}^\infty x(-3t+2) e^{-j\omega t} dt, \\ &= \int_{\infty}^{-\infty} x(u) e^{-j\omega \frac{(u-2)}{-3}} \frac{du}{-3}, \text{ (letting }u=-3t+2), \\ &= \frac{1}{3} \int_{-\infty}^\infty x(u)e^{-\frac{j\omega u}{-3}}e^{\frac{-2j\omega}{3}} du,\\ &= \frac{e^{\frac{-2j\omega}{3}}}{3}\int_{-\infty}^\infty x(u)e^{-\frac{j\omega u}{-3}} du, \\ &= \frac{e^{\frac{-2j\omega}{3}}}{3} \mathcal{X}(\frac{\omega}{-3}). \end{align} $

Question 5

a)

$ \mathcal{H}(\omega)=\mathfrak{F}\left\{e^{-3t}u(t)\right\}=\int_{-\infty}^\infty e^{-3t}u(t) e^{-j\omega t} dt = \int_{0}^\infty e^{-3t} e^{-j\omega t} dt =\left. \frac{e^{(-3-j\omega) t}}{ -3 -j \omega }\right|_0^\infty =\frac{1}{3+j\omega} $

b)

$ \mathcal{X}(\omega)=\mathfrak{F}\left\{e^{-2(t-2)}u(t-2)\right\}=\frac{e^{-2j\omega}}{2+j\omega} $

$ \begin{align} \mathcal{Y}(\omega)&=\mathcal{H}(\omega) \mathcal{X}(\omega) \\ &= \frac{e^{-2j \omega}}{(3+j\omega)(2+j\omega)} \\ &= \frac{e^{-2j\omega}}{6+5j\omega-\omega^2} \end{align} $

Now we need to find the partial fraction expansion of $ \mathcal{Y}(\omega) $.

Let

$ \frac{1}{(3+j\omega)(2+j\omega)}=\frac{A}{3+j\omega}+\frac{B}{2+j\omega} $

By multiplying both sides by $ (3+j\omega)(2+j\omega) $ and simplifying we get:

$ 2A+3B+(A+B)j\omega=1 $

After comparing, we get $ A=-1 $ and $ B=1 $.

Then,

$ \mathcal{Y}(\omega)=-\frac{e^{-2j \omega}}{(3+j\omega)}+\frac{e^{-2j \omega}}{(2+j\omega)} $

Now, using FT pairs and the time shift property, we have:

$ y(t)=e^{-2(t-2)}u(t-2)-e^{-3(t-2)}u(t-2) $

Question 6

a)

Taking the FT of both sides of the differential equation we get:

$ \begin{align} -\omega^2\mathcal{Y}(\omega)&=3j\omega\mathcal{Y}(\omega)-2\mathcal{Y}(\omega)+\mathcal{X}(\omega) \\ (-3j\omega-\omega^2+2)\mathcal{Y}(\omega)&=\mathcal{X}(\omega) \\ \mathcal{Y}(\omega)&=\frac{\mathcal{X}(\omega)}{2-3j\omega-\omega^2} \\ \end{align} $

Then,

$ \mathcal{H}(\omega)=\frac{1}{2-3j\omega-\omega^2} $.


b)

The frequency response of the system can be written as:

$ \mathcal{H}(\omega)=\frac{1}{(j\omega-1)(j\omega-2)} $.

Note that this can be done by either direct inspection or by finding the roots of the quadratic equation in the denominator.

Now, we need to find the partial fraction expansion of $ \mathcal{H}(\omega) $.

Let

$ \frac{1}{(j\omega-1)(j\omega-2)}=\frac{A}{j\omega-1}+\frac{B}{j\omega-2} $

By multiplying both sides by $ (j\omega-1)(j\omega-2) $ and simplifying we get:

$ -2A-B+(A+B)j\omega=1 $

After comparing, we get $ A=-1 $ and $ B=1 $.

Then,

$ \mathcal{H}(\omega)=-\frac{1}{j\omega-1}+\frac{1}{j\omega-2}=\frac{1}{1-j\omega}-\frac{1}{2-j\omega} $

Now, using linearity of FT, FT pairs, and time reverse property, we get:

$ h(t)=e^{t}u(-t)-e^{2t}u(-t) $ .




HW5

Back to 2011 Spring ECE 301 Boutin

Alumni Liaison

Questions/answers with a recent ECE grad

Ryne Rayburn